Set theory proof explanation (union of sets) [closed]












0












$begingroup$


I am practicing for my exam tomorrow and I stumbled upon an exercise I cannot solve



Prove that:
$cup A_i times cup B_j = cup(A_i times B_j)$
$i in I$, $j in J$,$(I,j) in Itimes J$



Can someone help me with that?










share|cite|improve this question











$endgroup$



closed as off-topic by José Carlos Santos, Eevee Trainer, Saad, Carl Schildkraut, Cesareo Dec 18 '18 at 10:35


This question appears to be off-topic. The users who voted to close gave this specific reason:


  • "This question is missing context or other details: Please provide additional context, which ideally explains why the question is relevant to you and our community. Some forms of context include: background and motivation, relevant definitions, source, possible strategies, your current progress, why the question is interesting or important, etc." – José Carlos Santos, Eevee Trainer, Saad, Carl Schildkraut, Cesareo

If this question can be reworded to fit the rules in the help center, please edit the question.












  • 1




    $begingroup$
    Show that any element of the LHS is in the RHS, and conversely. Post what you have done for a start and explain where you are stuck, then someone may be able to help you get a bit further. You will need to correct the typos in the question first.
    $endgroup$
    – David
    Dec 18 '18 at 0:14


















0












$begingroup$


I am practicing for my exam tomorrow and I stumbled upon an exercise I cannot solve



Prove that:
$cup A_i times cup B_j = cup(A_i times B_j)$
$i in I$, $j in J$,$(I,j) in Itimes J$



Can someone help me with that?










share|cite|improve this question











$endgroup$



closed as off-topic by José Carlos Santos, Eevee Trainer, Saad, Carl Schildkraut, Cesareo Dec 18 '18 at 10:35


This question appears to be off-topic. The users who voted to close gave this specific reason:


  • "This question is missing context or other details: Please provide additional context, which ideally explains why the question is relevant to you and our community. Some forms of context include: background and motivation, relevant definitions, source, possible strategies, your current progress, why the question is interesting or important, etc." – José Carlos Santos, Eevee Trainer, Saad, Carl Schildkraut, Cesareo

If this question can be reworded to fit the rules in the help center, please edit the question.












  • 1




    $begingroup$
    Show that any element of the LHS is in the RHS, and conversely. Post what you have done for a start and explain where you are stuck, then someone may be able to help you get a bit further. You will need to correct the typos in the question first.
    $endgroup$
    – David
    Dec 18 '18 at 0:14
















0












0








0





$begingroup$


I am practicing for my exam tomorrow and I stumbled upon an exercise I cannot solve



Prove that:
$cup A_i times cup B_j = cup(A_i times B_j)$
$i in I$, $j in J$,$(I,j) in Itimes J$



Can someone help me with that?










share|cite|improve this question











$endgroup$




I am practicing for my exam tomorrow and I stumbled upon an exercise I cannot solve



Prove that:
$cup A_i times cup B_j = cup(A_i times B_j)$
$i in I$, $j in J$,$(I,j) in Itimes J$



Can someone help me with that?







elementary-set-theory






share|cite|improve this question















share|cite|improve this question













share|cite|improve this question




share|cite|improve this question








edited Dec 18 '18 at 0:50









Andrés E. Caicedo

65.8k8160251




65.8k8160251










asked Dec 18 '18 at 0:09









mrnobodymrnobody

14818




14818




closed as off-topic by José Carlos Santos, Eevee Trainer, Saad, Carl Schildkraut, Cesareo Dec 18 '18 at 10:35


This question appears to be off-topic. The users who voted to close gave this specific reason:


  • "This question is missing context or other details: Please provide additional context, which ideally explains why the question is relevant to you and our community. Some forms of context include: background and motivation, relevant definitions, source, possible strategies, your current progress, why the question is interesting or important, etc." – José Carlos Santos, Eevee Trainer, Saad, Carl Schildkraut, Cesareo

If this question can be reworded to fit the rules in the help center, please edit the question.







closed as off-topic by José Carlos Santos, Eevee Trainer, Saad, Carl Schildkraut, Cesareo Dec 18 '18 at 10:35


This question appears to be off-topic. The users who voted to close gave this specific reason:


  • "This question is missing context or other details: Please provide additional context, which ideally explains why the question is relevant to you and our community. Some forms of context include: background and motivation, relevant definitions, source, possible strategies, your current progress, why the question is interesting or important, etc." – José Carlos Santos, Eevee Trainer, Saad, Carl Schildkraut, Cesareo

If this question can be reworded to fit the rules in the help center, please edit the question.








  • 1




    $begingroup$
    Show that any element of the LHS is in the RHS, and conversely. Post what you have done for a start and explain where you are stuck, then someone may be able to help you get a bit further. You will need to correct the typos in the question first.
    $endgroup$
    – David
    Dec 18 '18 at 0:14
















  • 1




    $begingroup$
    Show that any element of the LHS is in the RHS, and conversely. Post what you have done for a start and explain where you are stuck, then someone may be able to help you get a bit further. You will need to correct the typos in the question first.
    $endgroup$
    – David
    Dec 18 '18 at 0:14










1




1




$begingroup$
Show that any element of the LHS is in the RHS, and conversely. Post what you have done for a start and explain where you are stuck, then someone may be able to help you get a bit further. You will need to correct the typos in the question first.
$endgroup$
– David
Dec 18 '18 at 0:14






$begingroup$
Show that any element of the LHS is in the RHS, and conversely. Post what you have done for a start and explain where you are stuck, then someone may be able to help you get a bit further. You will need to correct the typos in the question first.
$endgroup$
– David
Dec 18 '18 at 0:14












1 Answer
1






active

oldest

votes


















0












$begingroup$

Glossing over trivialities by assuming $left( bigcup_{i in I} A_i right) times left( bigcup_{j in J} B_j right)$ is non-empty, here is one direction of the double containment argument:



Suppose $(x,y) in left( bigcup_{i in I} A_i right) times left( bigcup_{j in J} B_j right)$. By the definition of cartesian product we have $x in bigcup_{i in I} A_i$ and $y in bigcup_{j in J} B_j$. By the definition of set union we know there is $k in I$ so that $x in A_k$. Likewise there is $l in J$ so that $y in B_l$. And so we have $(x,y) in A_k times B_l$ with $(k,l) in I times J$. Therefore $(x,y) in bigcup_{(i,j) in I times J} left( A_i times B_j right)$.





It might be helpful to notice that the two hyperlinked definitions are fairly important here.






share|cite|improve this answer









$endgroup$




















    1 Answer
    1






    active

    oldest

    votes








    1 Answer
    1






    active

    oldest

    votes









    active

    oldest

    votes






    active

    oldest

    votes









    0












    $begingroup$

    Glossing over trivialities by assuming $left( bigcup_{i in I} A_i right) times left( bigcup_{j in J} B_j right)$ is non-empty, here is one direction of the double containment argument:



    Suppose $(x,y) in left( bigcup_{i in I} A_i right) times left( bigcup_{j in J} B_j right)$. By the definition of cartesian product we have $x in bigcup_{i in I} A_i$ and $y in bigcup_{j in J} B_j$. By the definition of set union we know there is $k in I$ so that $x in A_k$. Likewise there is $l in J$ so that $y in B_l$. And so we have $(x,y) in A_k times B_l$ with $(k,l) in I times J$. Therefore $(x,y) in bigcup_{(i,j) in I times J} left( A_i times B_j right)$.





    It might be helpful to notice that the two hyperlinked definitions are fairly important here.






    share|cite|improve this answer









    $endgroup$


















      0












      $begingroup$

      Glossing over trivialities by assuming $left( bigcup_{i in I} A_i right) times left( bigcup_{j in J} B_j right)$ is non-empty, here is one direction of the double containment argument:



      Suppose $(x,y) in left( bigcup_{i in I} A_i right) times left( bigcup_{j in J} B_j right)$. By the definition of cartesian product we have $x in bigcup_{i in I} A_i$ and $y in bigcup_{j in J} B_j$. By the definition of set union we know there is $k in I$ so that $x in A_k$. Likewise there is $l in J$ so that $y in B_l$. And so we have $(x,y) in A_k times B_l$ with $(k,l) in I times J$. Therefore $(x,y) in bigcup_{(i,j) in I times J} left( A_i times B_j right)$.





      It might be helpful to notice that the two hyperlinked definitions are fairly important here.






      share|cite|improve this answer









      $endgroup$
















        0












        0








        0





        $begingroup$

        Glossing over trivialities by assuming $left( bigcup_{i in I} A_i right) times left( bigcup_{j in J} B_j right)$ is non-empty, here is one direction of the double containment argument:



        Suppose $(x,y) in left( bigcup_{i in I} A_i right) times left( bigcup_{j in J} B_j right)$. By the definition of cartesian product we have $x in bigcup_{i in I} A_i$ and $y in bigcup_{j in J} B_j$. By the definition of set union we know there is $k in I$ so that $x in A_k$. Likewise there is $l in J$ so that $y in B_l$. And so we have $(x,y) in A_k times B_l$ with $(k,l) in I times J$. Therefore $(x,y) in bigcup_{(i,j) in I times J} left( A_i times B_j right)$.





        It might be helpful to notice that the two hyperlinked definitions are fairly important here.






        share|cite|improve this answer









        $endgroup$



        Glossing over trivialities by assuming $left( bigcup_{i in I} A_i right) times left( bigcup_{j in J} B_j right)$ is non-empty, here is one direction of the double containment argument:



        Suppose $(x,y) in left( bigcup_{i in I} A_i right) times left( bigcup_{j in J} B_j right)$. By the definition of cartesian product we have $x in bigcup_{i in I} A_i$ and $y in bigcup_{j in J} B_j$. By the definition of set union we know there is $k in I$ so that $x in A_k$. Likewise there is $l in J$ so that $y in B_l$. And so we have $(x,y) in A_k times B_l$ with $(k,l) in I times J$. Therefore $(x,y) in bigcup_{(i,j) in I times J} left( A_i times B_j right)$.





        It might be helpful to notice that the two hyperlinked definitions are fairly important here.







        share|cite|improve this answer












        share|cite|improve this answer



        share|cite|improve this answer










        answered Dec 18 '18 at 1:02









        Matt A PeltoMatt A Pelto

        2,622621




        2,622621















            Popular posts from this blog

            Plaza Victoria

            In PowerPoint, is there a keyboard shortcut for bulleted / numbered list?

            How to put 3 figures in Latex with 2 figures side by side and 1 below these side by side images but in...